Answer (C) is correct . Since fixed costs, unit sales, and sales price are projected to remain the same, and operating income will be reduced, the variable costs must increase. If the July Year 2 prediction that unit sales will increase by 10% from 150,000 to 165,000 is based on the budgeted FC, sales price, and VC, predicted operating income can be
Answer (A) is incorrect because This amount is the projected net income before the increase in sales. Answer (B) is incorrect because The amount of $330,000 takes into account a 10% increase in fixed costs to $990,000 when the fixed costs did not increase. Answer (D) is incorrect because The UCM for year 2 is $8 per unit, not $9.
|